Which of the following statements is most likely true about the members of a network family?

If you're seeing this message, it means we're having trouble loading external resources on our website.

If you're behind a web filter, please make sure that the domains *.kastatic.org and *.kasandbox.org are unblocked.

 . Question 32 (2 points) Given that 8 employees have the following Promotion possibilities: Lower, Middle-A, Middle-B, Upper-C, Upper-B, Upper-A, Supervisor, Executive, Manager, AVP, SVP, VP, Presid...

BonesNoBones.csv: "Date","BonesDay","Day_of_Week","Weather_NYC" "11/11/2021","No Bones Day","Thursday","Cloudy" "11/09/2021","Bones Day","Tuesday","Sunny" "11/08/2021","Bones Day","Monday","Sunny" "11...

BonesNoBones.csv: "Date","BonesDay","Day_of_Week","Weather_NYC" "11/11/2021","No Bones Day","Thursday","Cloudy" "11/09/2021","Bones Day","Tuesday","Sunny" "11/08/2021","Bones Day","Monday","Sunny" "11...

BonesNoBones.csv: "Date","BonesDay","Day_of_Week","Weather_NYC" "11/11/2021","No Bones Day","Thursday","Cloudy" "11/09/2021","Bones Day","Tuesday","Sunny" "11/08/2021","Bones Day","Monday","Sunny" "11...

BonesNoBones.csv: "Date","BonesDay","Day_of_Week","Weather_NYC" "11/11/2021","No Bones Day","Thursday","Cloudy" "11/09/2021","Bones Day","Tuesday","Sunny" "11/08/2021","Bones Day","Monday","Sunny" "11...

BonesNoBones.csv: "Date","BonesDay","Day_of_Week","Weather_NYC" "11/11/2021","No Bones Day","Thursday","Cloudy" "11/09/2021","Bones Day","Tuesday","Sunny" "11/08/2021","Bones Day","Monday","Sunny" "11...

MCAT Content / AAMC MCAT Practice Exam 1 Ps Solutions

AAMC FL1 PS [Web]

Exam 1 P/S Solutions: Passage 1

1) First thing we’re going to do as we jump into this question is differentiate between the two conditions. The author tells us there were two different types of training conditions. “The participants in the simultaneous condition were presented with all the training objects all at once and were asked to study their locations simultaneously. The participants in the serial condition were presented with the training objects one at a time.” We’re focused on the serial condition and how it would differ from the simultaneous condition. Typically, a setup like the serial condition can lead to the serial-position effect, which is the tendency of a person to recall the first and last items in a series best, and the middle items worst.

  1. a primacy effect. In free recall, there is often a primacy effect in which the items that were presented first are recalled well as they have already been encoded into long-term memory. This is consistent with the serial-position effect we mentioned in the breakdown. This is a strong answer. 
  2. a state dependency effect. This is the phenomenon where people will learn something in a particular state, they will recall information better in that same state. For example, if you learn a behavior on a particular drug, you’ll perform that behavior better in the future on that same drug. Not what’s happening in the serial condition. 
  3. a misinformation effect. The misinformation effect is an example of retroactive interference. Retroactive interference occurs when old memories are changed by new ones, sometimes so much that the original memory is forgotten. Misinformation effect happens when a person’s recall of memories becomes less accurate because of additional information following the event. This is not more likely in the serial condition of the experiment 
  4. a dual-coding effect. Verbal and visual information are processed differently. By utilizing both while learning, information can be learned and internalized better. This is also not more likely in the serial condition of the experiment. We can stick with answer choice A as our best option. 

2) To answer this question, we want to make sure to visualize what is going on in the question stem. There’s a slight change to the experimental procedure where the participants now have to recall all the objects instead of placing them in a box. Let’s define spreading activation and give an example. Spreading activation is a theory proposed that relates storage of memories to the activation of a series of nodes. These nodes create an activation pattern of other related nodes and this is how singular events are remembered. For example, at the mention of the color red and the word “vehicle,” two nodes that may be activated at the same time to relate a personal memory of a car they owned, or a fire truck.

  1. Making source monitoring errors regarding the location of the training objects A source-monitoring error is a memory error in which the source of a memory is incorrectly attributed to some specific recollected experience. This possible error is not relevant to this specific question or procedure.
  2. Poorer memory for the training objects seen at the later points in the experiment Recency effect is when the items that were presented last are also recalled well as they are still being held in the working short-term memory. This is what we expect to happen here as well. We don’t expect poorer memory for the training object at the later points in the experiment.
  3. Selective forgetting of the training objects that were placed in the center of the box Selective forgetting involves recalling things that are more consistent with the participant’s own thoughts and ideas. This is not relevant to this specific experiment. 
  4. Recalling objects that were not presented but are from the same category as the training objects This is consistent with the spreading of activation theory. The participants might recall objects from the same category as the training objects because of the activation pattern in the participants’ minds. This is a likely memory error in this procedure, so we can stick with answer choice D as our best answer.

3) This is an open-ended question so we’ll dive right into the four answer choices in a second. We want to note the results of the experiment were found in Figure 1.

The central placement error was determined based on the errors participants made in placing objects from the same category nearer to the central point than their original locations. Looking at the serial condition, Adults made greater average central placement errors than the younger groups. 

  1. is comparable to adults by nine years of age. We can look at Figure 1 and note that in the serial condition, adults had significantly greater average central placement error. Memory for locations is not comparable.
  2. is influenced by categorical knowledge. This is consistent with what we read in the passage and what we saw in the results. Adults have greater categorical knowledge, and that ultimately influenced the memory for locations. Average central placement error was greater in adults.
  3. becomes more accurate with age. This is the opposite of what we see in Figure 1. Adults made greater average central placement errors than the younger groups.
  4. relies solely on recall of distances. The author explains that memory for locations was also influenced by categorical bias. If categorical bias didn’t exist, it’s possible adults would not have had central placement error. We can stick with answer choice B as the best answer choice. 

4) Piaget’s theory of cognitive development in childhood focused on four key stages from birth to young adulthood. It’s crucial you know these for test day and the corresponding ages and details. The concrete operational stage is characterized by the appropriate use of logic and when children learn the idea of conservation. This corresponds to ages 7-11 years old, so some of the children are at the later end of this range, while some are just entering this range. The liquid volume example is a classic one used by instructors. Younger children will not state that the water is the same volume because of the different shape of the beaker.

  1. All three age groups will state that the water in the taller beaker is greater in quantity. This is not expected. The concrete operational stage corresponds to ages 7-11 years old. As we mentioned, some of the children are at the later end of this range, while some are just entering this range. It’s expected the older children will realize the amount of water in both beakers is the same and only the shape of the beakers is different.
  2. The majority of the 11-year-olds will state that the amount of water in the taller beaker is the same as in the original beaker. This is consistent with our breakdown and what we know about the concrete operational stage. 11-year olds are at the later end of the 7-11 year old range, so they are likely to understand the amount of water in both beakers is the same. 
  3. The majority of the 9- and 11-year-olds will state that the amount of water in the taller beaker is greater in quantity. This is inconsistent with what we know about the concrete operational stage. The older kids will understand the amount of water in both beakers is the same. 
  4. The majority of the 7-year-olds will state that the amount of water in the taller beaker is the same as in the original beaker. This is more likely to happen with the majority of the 11-year olds. The 7-year olds are just entering the concrete operational stage, so they will believe the amount of water in the taller beaker is greater in quantity. Answer choice B is going to remain our best answer. 

Exam 1 P/S Solutions: Passage 2

5) To answer this question, we can go back to the passage and note what the author mentions about Maintenance Factor 1. Then, we’ll go through the four answer choices and define each option to find the best answer choice.

We’re dealing with classical conditioning here. Classical conditioning is a form of learning whereby a conditioned stimulus becomes associated with an unrelated unconditioned stimulus to produce a behavioral response known as a conditioned response. 

  1. Psychodynamic. Psychodynamic theory relates childhood events to our adult lives. These events will shape our personalities. This is not what we’re seeing in Maintenance Factor 1.
  2. Behaviorist. A behaviorist considers actions as being responses to external stimuli. The change in bodily sensation produces a behavioral response in this situation: that’s the fear being experienced by these patients.  
  3. Trait. Trait theorists believe personality can be conceptualized as a set of common traits, or characteristic ways of behaving that every individual exhibits to some degree. This is also out of scope here. Answer choice B remains the superior answer.
  4. Humanistic. Humanistic perspective argues that an individual’s subjective free will is the most important determinant of behavior. The conditioned fear is not consistent with this approach. Answer choice B is going to be our best answer.

6) This question ties into our previous question. Diving into every passage and knowing the significance of everything the author presents us is key to really understanding the passage. We looked at Maintenance Factor 1, which was conditioned fear in response to changes in bodily sensations. We said we’re dealing with classical conditioning here. Classical conditioning is a form of learning whereby a conditioned stimulus becomes associated with an unrelated unconditioned stimulus to produce a behavioral response known as a conditioned response. The change in bodily sensation (conditioned stimulus) produces fear and panic (conditioned response) in this situation.

  1. unconditioned stimuli. An unconditioned stimulus will automatically trigger a response. For example, the smell of a pizza may make someone hungry.
  2. conditioned stimuli. This is exactly what the change in bodily sensations act as. Once the patients associated mild changes in bodily sensation with a panic attack, these changes eventually stimulated the panic attack. The mild changes in bodily sensations act as conditioned stimuli and elicit the conditioned fear and panic response.
  3. unconditioned responses. The unconditioned response is the panic attack. It is the natural, unlearned behavior to a given stimulus.
  4. conditioned responses. The conditioned response is the fear and panic that comes along with panic attacks. Answer choice B is going to be the best answer choice to describe the mild changes in bodily sensations.

7) To answer this question, we can go back to the passage. We can pull up the response of Patient 2 to the question, “Do you worry about the panic attacks that you experience?”

Patient 2 is so terrified of having a panic attack that they avoid meetings altogether. They are worried about others noticing their nervousness as well. We can dive into the four key words listed as answer choices and find the one that best describes the panic attacks of Patient 2.

  1. discriminatory stimuli. This seems like a distractor. A discriminative stimulus is used to get a specific response consistently in the presence of a stimulus. For example, a dog might sit as soon as it sees its owner because the dog knows that the owner often gives it treats. Alternatively, stimulus discrimination involves being able to differentiate between a specific stimulus and similar stimuli. Neither one is applicable to Patient 2.
  2. signaling stimuli. This is another distractor. A signaling stimulus is not something AAMC lists on the content outline. We are not going to pick this answer choice as it is out of scope.  
  3. positive punishers. In the context of operant conditioning, whether you are reinforcing or punishing a behavior, “positive” always means you are adding a stimulus (not necessarily a good one). Positive punishments add an aversive stimulus to decrease a behavior or response. In this case, the worry and dread are being added, while the punishment is not going to meetings any longer.
  4. negative punishers. This ties into answer choice C as “negative” always means you are removing a stimulus (not necessarily a bad one). Negative reinforcers remove an aversive or unpleasant stimulus to increase or maintain the frequency of a behavior. We are not removing a stimulus in this case, we’re actually seeing the addition of worry and dread. Answer choice C is going to be our best answer.

8) To answer this question, we can go back to the passage and note what the author says about interoceptive awareness. High interoceptive awareness is a heightened ability to detect bodily sensations of arousal, such as increases in heartbeat or blood pressure. These are involuntary changes, so at first glance, I am thinking autonomic nervous system or sympathetic nervous system. The autonomic nervous system controls the workings of internal organs such as the heart, lungs, digestive system, and endocrine systems; it does so without conscious effort. The sympathetic nervous system controls the body’s automatic response to danger, increasing the heart rate, dilating the blood vessels, slowing digestion, and moving blood flow to the heart, muscles, and brain.

  1. reticular activation system. The reticular activating system is a necessary part in the processing of emotion. It is believed to first arouse the cortex and then maintain its wakefulness so that sensory information and emotion can be interpreted more effectively. This is not relevant to the increased heartbeat and blood pressure.
  2. autonomic nervous system. This is something I covered in the breakdown of the question. The autonomic nervous system controls the workings of internal organs such as the heart, lungs, digestive system, and endocrine systems; it does so without conscious effort. The increases in heartbeat and blood pressure are tied to the activity of the autonomic nervous system. This is going to be our best answer so far.
  3. limbic system. The limbic system is the area of the brain most heavily implicated in emotion and memory. Its structures include the hypothalamus, thalamus, amygdala, and hippocampus. This is out of scope in this question as well.
  4. somatic nervous system. The somatic nervous system is the part of the peripheral nervous system that transmits signals from the central nervous system to skeletal muscle, and from receptors of external stimuli to the central nervous system, thereby mediating sight, hearing, and touch. This does not include the increased heartbeat or blood pressure. Answer choice B remains the more relevant and correct answer. 

9) This is something we’ve touched on tangentially as we’ve gone through the other questions in this set. We can recall the details we were given about the two maintenance factors, then we can relate them to the answer choices. Maintenance Factor 1 is conditioned fear in response to changes in bodily sensations. Maintenance Factor 2 is the misappraisal of bodily sensations such as interpreting bodily sensations as signs of imminent death or loss of control. 

For Maintenance Factor 1, when we went through Question 6 we said we’re dealing with classical conditioning. Classical conditioning is a form of learning whereby a conditioned stimulus becomes associated with an unrelated unconditioned stimulus to produce a behavioral response known as a conditioned response. The change in bodily sensation (conditioned stimulus) produces fear and panic (conditioned response) in this situation. Right away, this narrows our possible answer choices down to either B or D. 

For Maintenance Factor 2, the misappraisal of bodily sensations is a cognitive process. Cognitive appraisal is the interpretation of a situation that influences the extent to which a situation is perceived as stressful. It’s subjective, so bodily sensations can be interpreted as signs of imminent death, loss of control, or even positives in some cases. We narrowed our answer choices down to B or D previously, now we can confirm answer choice D is going to be our best option.

  1. Operant conditioning and top-down processes Operant conditioning is a type of associative learning process through which the strength of a behavior is modified by reinforcement or punishment. Top-down processing refers to how we interpret sensations due to influences from our available knowledge, our experiences, and our thoughts
  2. Classical conditioning and affective processes Affective processes include feelings and responses to emotion-laden behavior.
  3. Operant conditioning and physiological processes Physiological processes are physical and not cognitive.
  4. Classical conditioning and cognitive processes This answer choice matches our breakdown. We’re sticking with answer choice D as our best answer.

10) To answer this question, we can break down what we know about Maintenance Factor 1. We’re told patients with high interoceptive awareness are likely to associate mild changes in bodily sensations (such as slightly heightened heartbeat) with the panic they experienced during panic attacks. This association leads to conditioned fear in response to changes in bodily sensations (Maintenance Factor 1). We’re going to describe and analyze our four answer choices. Three of them are types of research from which we can get data on the role of Maintenance Factor 1 on PD. Our correct answer is going to be the one from which this data is LEAST likely to come.

  1. Correlational studies. Correlational research involves measuring variables and their relationship. These can be used to relate panic attacks, conditioned fear, and other factors. 
  2. Case studies. A case study is used to analyze a principle or thesis. Detailed considerations is given to a specific situation or person over time. This would be helpful in determining the effects of panic attacks and the conditioned fear.
  3. Longitudinal studies. Longitudinal research describes a research study that follows the population or group of interest at multiple time points. This would be helpful as the effects of the panic attacks and conditioned fear can be tracked.
  4. Experimental studies. Experimental research is research in which scientists manipulate variables to test the difference between an experimental and control group. Experimental studies can be very helpful, but in a case like this, they would have to manipulate the physiological states of the participants in the studies which is not done simply and readily. Answer choice D is going to be our best answer.

Exam 1 P/S Solutions: Questions 11-14

11) This is a standalone question that is going to require us to visualize what is going on. The test-maker describes a rare genetic disorder that involves flaking skin behind the eyelids. That flaking skin can damage the eye, but we want to know specifically which structure is most susceptible to damage. 

Which of the following statements is most likely true about the members of a network family?

  1. Choroid. Referencing the figure, the choroid is the layer between the retina and the sclera. We can see it is likely not the structure that is most at risk.
  2. Cornea The cornea is the transparent layer forming the front of the eye. If we reference our visual, this is going to be what we’re most worried about. The flaking skin can damage the cornea.
  3. Lens. The lens is a transparent biconvex structure in the eye that, along with the cornea, helps to refract light to be focused on the retina. The lens is not in direct danger, but rather the cornea is still at highest risk of being damaged.
  4. Retina. The retina is the thin layer of cells at the back of the eyeball where light is converted into neural signals sent to the brain. The back of the eyeball would not be affected like the cornea at the front of the eye. We can stick with answer choice B as our best answer.

12) This is a standalone question that relies on our knowledge of the ear. Hair cells are the sensory receptors of both the auditory system and the vestibular system in the ears of all vertebrates. When sound waves produce fluid waves inside the cochlea, the basilar membrane flexes, bending the stereocilia that attach to the tectorial membrane. Their bending results in action potentials in the hair cells, and auditory information travels along the neural endings of the bipolar neurons of the hair cells to the brain. All of the stereocilia are mechanoreceptors, and when bent by vibrations they respond by opening a gated ion channel. As a result, the hair cell membrane is depolarized, and a signal is transmitted to the chochlear nerve. 

  1. Chemically. Chemically-gated channels are usually found on dendrites. Like the name suggests, they open and close because of different chemicals. That includes neurotransmitters, hormones, and ions. 
  2. Mechanically. We mentioned in our breakdown, stereocilia are mechanoreceptors that respond by opening a gated ion channel. That’s exactly what we’re looking for to answer this question. We can keep answer choice B as our best answer. 
  3. Electrically. We do not haven electrically gated channels, but rather mechanically gated channels.  
  4. Synaptically. This answer choice is a distractor. Synaptically-gated ion channels are not one the 3 main types of gated channels. Neurotransmitters will diffuse across the synaptic cleft to bind to ligand-gated ion channels, but that is not the case in our example. We can stick with answer choice B as our best answer. Sound-induced vibrations depolarize hair cells of cochlea by opening mechanically-gated ion channels. 

13) My first instinct is that I am not a fan of Max’s behavior here! However, I’ll put that aside as we’re looking at it according to the actor-observer bias. Actor-observer bias is where an actor tends to explain his or her behavior by situational factors, while an observer tends to explain the actor’s behavior based on stable, internal states. We have an external vs. internal explanation for behavior.

A.

Which of the following statements is most likely true about the members of a network family?

This is consistent with our breakdown. Max’s self-judgment is going to be attributed to situational factors (like not feeling well). Sam’s judgment is attributed to internal factors (like Max being socially awkward).

B.

Which of the following statements is most likely true about the members of a network family?

This is the opposite of what we want in an answer choice. Max’s self-judgment is going to be attributed to situational factors (like not feeling well). Sam’s judgment is attributed to internal factors (like Max being socially awkward). This presents those views backwards.

C.

Which of the following statements is most likely true about the members of a network family?

Only the “observer” is going to attribute the actor’s behavior to internal factors. Max would attribute his behavior to not feeling well (external or situational).

D.

Which of the following statements is most likely true about the members of a network family?

This is going to be similar to answer choice C. Only the “actor” will attribute their behavior to situational factors. Sam would attribute Max’s behavior to Max being socially awkward (internal). 

14) This is a standalone question in which a researcher administers a stimulus to the participant’s tongue. Eventually the response to the stimulus decreases, and the researcher switches to a new stimulus. We’re going to define all of our four answer choices and find the one that describes the process taking place. 

  1. Sensory perception. Sensory perception involves using our senses to process the stimuli in the environment. The processing occurs through our senses. This is not what’s happening in the question stem. The researcher is not testing the sensory perception of the participant. 
  2. Habituation and dishabituation. Habituation and dishabituation are types of nonassociative learning where habituation involves the diminished response to a frequently repeated stimulus while dishabituation is the fast recovery of a response that has undergone habituation. Bingo! This is exactly what is taking place in the question stem. We have habituation (lemon juice) followed by dishabituation (lime juice). 
  3. Stimulus generalization in classical conditioning. Classical conditioning occurs when an appropriate response to a stimulus becomes conditioned to respond to another associated stimulus. Stimulus generalization is demonstrating the conditioned response to stimuli that are similar to the conditioned stimulus. However, in this case we don’t know if classical conditioning is happening.  
  4. Conditioned responses in classical conditioning. Conditioned responses are responses caused by the conditioned stimulus. Conditioned responses are learned responses, but salivation is an unconditioned response. We can stick with answer choice B as our best answer. 

Exam 1 P/S Solutions: Passage 3

15) We can approach this question by first defining the optimal arousal theory, then we’ll lookin into the results of Study 1. Emotion can determine an individual’s performance in learning. Theories of learning assert that there is an optimal level of arousal (stress) that we all try to maintain.  If we are under-aroused, we become bored and will seek out some sort of stimulation.  On the other hand, if we are over-aroused, we will engage in behaviors to reduce our arousal/stress.

Which of the following statements is most likely true about the members of a network family?

In the passage, Study 1 was used to investigate the math performance of Asian-American women.

The different groups were subject to reminders of different stereotypes. The gender identity group’s performance suffered, while the ethnic identity group’s performance improved. We want to relate what we can see from these results to one of the 4 answer choices. 

  1. their membership in a stereotyped group leads to optimal arousal, which results in varying levels of performance depending on ability. Optimal arousal would theoretically yield the best results on the math test. However, reminding the gender identity group of their membership to the stereotyped group led to less-than-optimal arousal and performance most likely suffered. The results did not simply vary because of ability.
  2. their membership in a stereotyped group leads to fluctuating arousal and results in poor performance. This is not the case. By reminding the group of their ethnic identity, performance actually increased. It’s the negative stereotype that really increased arousal past the optimal level and led to poor performance.
  3. a negative stereotype associated with their identity increases arousal beyond what is optimal and leads to poor performance. This answer choice matches what we see in the passage. The gender identity group had to answer personal questions related to their gender identity. That increased arousal, but so much so that the average percent correct on the math test was comparatively very low. This is going to be our best answer choice so far.
  4. a positive stereotype associated with them eliminates arousal completely, which results in optimal performance. This is not true as an extremely low level of arousal can lead to boredom or apathy. That will hinder performance as well. We can stick with answer choice C as our best option. 

16) To answer this question, we will once again need to go back to the passage to get some necessary information. The passage tells us Asian Americans are stereotyped as having high math ability. Therefore, the participants that identify as Asian Americans more strongly and make being an Asian American a central part of their self-concept will perform better. The participants that do not make being an Asian American a central part of their self-concept will not get that same boost. 

  1. be more affected by the gender stereotypes regarding their math performance. This is out of scope here. The question stem does not address gender, and the passage does not relate both ethnicity and gender together at once. We cannot say this answer choice is likely. 
  2. not be affected by the negative stereotype regarding women’s math ability. This is similar to answer choice A. We cannot say with any level of certainty whether this is likely or not. The passage did not tell us about gender and ethnic identities effects together. We also don’t know if someone whose ethnic identity was a central part of their self-concepts doesn’t also strongly relate to their gender.
  3. be more affected by the positive stereotype regarding Asian Americans’ math ability. This answer choice is consistent with our breakdown. If someone identifies strongly as an Asian American, they would be more affected by the positive stereotype regarding Asian Americans’ math ability. This is going to be the best answer so far. 
  4. perform the same regardless of which condition they were placed in. The entire passage talked about how stereotypes associated with a social group can affect performance. We saw results that were consistent with this as well where different experimental conditions yielded different math scores. It’s unlikely these participants would perform the same regardless of which condition they were placed in. Answer choice C is going to be our best answer.

17) The sympathetic nervous system activates what is often termed the fight or flight response. The sympathetic nervous system controls the body’s automatic response to danger, increasing the heart rate, dilating the blood vessels, slowing digestion, and moving blood flow to the heart, muscles, and brain. We’re going to analyze the four measures in the answer choices and decide which would be most useful to measure the degree of sympathetic arousal.

  1. A measure of electrical conductivity of the skin. Typically, an involuntary increase in sweating increases skin conductivity, which is an indication of psychological and physiological arousal. This is going to be a great way for researchers to look into the degree of sympathetic arousal.
  2. A CT scan of the hindbrain. This is going to give us a detailed image of the hindbrain. The hindbrain is the portion of the brain that contains the pons, cerebellum, and medulla, and is responsible for regulating basic human functions. Answer choice A is going to be a better way to measure.
  3. A PET scan of the parietal cortex. This is going to be similar to answer choice B. PET scans measure the binding of a radioactive “tracer” molecule in the brain. The parietal lobes function in sensation and perception, and integrating sensory input. Answer choice A remains the superior answer.
  4. A measure of melatonin levels. Melatonin is a hormone that regulates the sleep-wake cycle. This is also out of scope, so we can stick with answer choice A as the best answer. 

18) This is something I like making sure I try and figure out as I’m reading the passage. Why is that? Because this could be a question in a huge portion of the passages you’ll see on the exam. It’s imperative you understand the experiments and studies you come across, because the test-maker can (and will) always give you questions that show you understand the purpose, design, variables, results, and conclusions of any study or experiment. If we have experimental results, we want to be able to make conclusions from the results and any data. For this particular question, we can go through the four answer choices and see which conclusion is supported by Figure 1. 

We can make some general observations here. Group 1 showed the lowest average percent correct on the math test. This was the group where they played into the negative stereotype. Group 1 had the highest score and had the positive stereotype. Group 3 is the control group. 

  1. Priming stereotypes always has a negative impact on performance. We can see this is not true by looking at the Ethnic identity group. The stereotype actually had a positive impact on performance. This conclusion is not supported by Figure 1. 
  2. Priming a negative stereotype interferes with performance. This is consistent with what we see in the gender identity group. Average percent correct is lower than the other groups.
  3. Priming a positive stereotype does not affect performance. This is inconsistent with what we see in Figure 1. The Ethnic identity group actually saw an increase in average percent correct. We can eliminate this answer choice.
  4. Priming stereotypes does not affect performance as much as one’s self-concept. This is not something that was explored in this part of the study or shown in Figure 1. We cannot make this conclusion based on what we see in Figure 1. We can stick with answer choice B as our best option.

Exam 1 P/S Solutions: Passage 4

19) This is a fairly typical question setup from AAMC. We go through a passage and an experimental setup, then the test-maker will change something about the experiment to really make sure we understand what is going on. In this case, we’re going to have to consider in-group and out-group dynamics. 

In sociology and social psychology, in-groups and out-groups are social groups to which an individual feels as though he or she belongs as a member, or towards which they feel contempt, opposition, or a desire to compete, respectively. In-group bias is a phenomenon in which people tend to act more favorably towards people who they perceive to be part of their in-group. In-group bias becomes more extreme in times of conflict. We know from the passage the participants who had been in the stress condition were more likely to trust the other player than those who had been in the control condition.

A.

Which of the following statements is most likely true about the members of a network family?

There is no reason for the control group participants to trust the rival university student more than the same university student. We do know from the passage that the participants who had been in the stress condition were more likely to trust the other player than those who had been in the control condition. We can look for a better answer choice.

B.

Which of the following statements is most likely true about the members of a network family?

This answer choice incorrectly shows the likelihood of trusting the other player as being roughly the same. We know this is not the case. For starters, the participants who had been in the stress condition were more likely to trust the other player than those who had been in the control condition. Presumably the students would trust people from the same university more than they would students from a rival university.

C.

Which of the following statements is most likely true about the members of a network family?

This answer choice is consistent with what we read in the passage. We know the participants who had been in the stress condition were more likely to trust the other player than those who had been in the control condition. That’s why we see the stress condition have higher graphs. Additionally, students trust students from their same university and not the rival university. This is our best answer so far.

D.

Which of the following statements is most likely true about the members of a network family?

This answer choice correctly shows a higher likelihood of trusting other players in the stress condition. However, it does not consider the different universities. We expect students will trust the players that came from the same university. 

20) This ties into our previous question as well. Knowing the ins and outs of these studies really helps you understand the big picture in every passage. This is something that can be used in every section of the exam. There are typically independent and dependent variables in every study or experimental passage. We need to identify both, and more importantly, we have to understand their relationship. An independent variable, just like its name suggests, can be controlled or changed during a study-this variable(s) is independent of the other factors of the experiment. As the independent variable is changed, we’re tracking the dependent variable to see the effect of the change in the independent variable. So, the independent variable varies, and we track the change on the dependent variable. In the case of this experiment, the independent variable is the exposure to stress. One group had to give a presentation (a social stressor), while the control group did not have to give a public presentation. We expect the stress group would have higher heart rate and skin conductance. 

  1. for all the participants before they participate in the first phase of the experiment compared to after. This is not likely in the control group. We only expect the stress group would have higher heart rate and skin conductance. 
  2. for the participants who chose to share $120 compared to those who chose to walk away with $30. The independent variable was the exposure to the stressor. We would not be able to verify it had its intended group if we’re looking at all the participants that chose to share $120. We have to make sure to keep in mind what the question is asking specifically. 
  3. for the participants in the stress group than for the participants in the control group. This is consistent with what we know from the passage. We expect the stress group would have higher heart rate and skin conductance. This verifies the independent variable had the intended effect because the public presentation caused a higher heart rate and skin conductance in the stress group. 
  4. for all the participants before they participate in the second phase of the experiment compared to after. This is another answer choice that is not going to help us verify the independent variable has its intended effect. Knowing the independent variable was a big part of answering this question correctly. We can stick with answer choice C as our best option.

21) Before we jump into the 4 statements, we can revisit the passage for some context. 

We know three of the statements will provide a likely explanation for the researchers dropping the highly anxious participants, while the correct answer will provide the LEAST likely explanation. 

  1. Ethical committees do not allow research with participants who may have psychological disorders. Research with participants who may have psychological disorders is allowed and often done on purpose. There should not be any harm done to the participants, but research is allowed. This is an unlikely explanation, but we still want to go through the additional answer choices to make sure this is the LEAST likely.
  2. If all the anxious participants get assigned to the stress condition, this may pose a confounding variable. A confounding variable is a variable that can influence the results but is not considered. This is a great reason to not include the highly anxious participants. The researchers do not want anything to interfere with their results.
  3. Highly anxious participants are not representative of the population and may reduce the generalizability of the findings. This is also a great reason to exclude the five highly anxious participants from the sample. The research group hypothesized that when exposed to stressors, people will be more inclined to engage in prosocial behaviors as a coping mechanism. If we have participants that are not representative of the results, we can reduce the generalizability which is not what we want from any research.
  4. If all the anxious participants get assigned to the control condition, this may lead to a false rejection of the experimental hypothesis. This is also a possibility. The research group hypothesized that when exposed to stressors, people will be more inclined to engage in prosocial behaviors as a coping mechanism. By including the outliers, we may incorrectly reject the hypothesis. We can stick with answer choice A as the least likely explanation and our correct answer. 

22) This is another question that deals with adding something to the experiment given to us in the passage. AAMC loves doing this to make sure we understand the studies and experiments we come across. 

Selye defined a series of symptoms in experiments with rats as General Adaptation Syndrome. It consists of three stages: the alarm stage, the resistance stage, and the exhaustion stage.

Which of the following statements is most likely true about the members of a network family?

The alarm stage is similar to the fight-to-flight response, and the body mobilizes resources to react to the incoming noxious agent. The resistance forces will be built up when the noxious challenge is detected as continuing. The exhaustion stage will cause death if the body is unable to overcome the threat. The theory demonstrated that stress that is perceived as a threat (distress) may be debilitating if it is continuous. 

  1. necessary, because humans respond differently to different types of stressors. This change is not necessary. Humans will respond similarly to different types of stressors and the stressor will initiate the body’s response. There won’t be different responses to different types of stressors.
  2. necessary, because physical stressors cause avoidance–avoidance conflict, whereas social stressors cause approach–approach conflict. Avoidance-avoidance conflict means picking between two equally undesirable choices. Approach-approach conflict means deciding between two appealing goals. This answer choice is incorrect because humans will respond similarly to different types of stressors.
  3. unnecessary, because the human stress response is not specific to the type of stressor. This is correct and something I touched on in answer choice A. Humans will respond similarly to different types of stressors. The stressor will initiate the body’s response and the response will not vary based on the type of stressor. This is going to be our best answer so far.
  4. unnecessary, because both physical stressors and social stressors cause avoidance–avoidance conflict. Avoidance-avoidance conflict means picking between two equally undesirable choices. Selye’s theory does not talk about conflict types, but rather mentions that humans will respond similarly to different types of stressors and the stressor will initiate the body’s response. We can stick with answer choice C as our best option.

Exam 1 P/S Solutions: Passage 5

23) This is something we always want to pay attention to as we read the passage. Why is that? Because this could be a question in a huge portion of the passages you’ll see on the exam. It’s imperative you understand the experiments and studies you come across, because the test-maker can (and will) always give you questions that show you understand the purpose, design, variables, results, and conclusions of any study or experiment. If we have a study, we want to be able to make conclusions from the results or any observations. We were told in Study 2, “The results supported the researchers’ hypothesis.” We know Study 2 was testing the hypothesis that, “synchronous actions make participants more prone to comply when prompted to engage in aggression.” We’re going to find a conclusion supported by the outcome:

  1. Synchronous activity increases group polarization. Group polarization is the tendency for groups to make decisions that are more extreme than the initial inclination of its members. In this case, there was no initial inclination that is made more extreme. This is a decent answer for now. 
  2. Synchronous activity increases conformity. Conformity is the act of matching attitudes, beliefs, and behaviors to group norms. This is consistent with the hypothesis. Study 2 was testing the hypothesis that, “synchronous actions make participants more prone to comply when prompted to engage in aggression.” We also know the results supported this hypothesis. This answer is going to be superior to answer choice A. 
  3. Asynchronous activity increases cognitive dissonance. In psychology, “cognitive dissonance” describes the mental stress or discomfort experienced by an individual who holds two or more contradictory beliefs, ideas, or values at the same time, or is confronted by new information that conflicts with existing beliefs, ideas, or values. 
  4. Asynchronous activity increases social loafing. Social loafing is the tendency for people to put forth less effort when working on a group task if the individual contributions aren’t evaluated. Social loafing can be reduced by making the task more challenging, separating the performance of individuals in the group, or making each individual’s performance essential for success. Answer choice B is going to be our correct answer.

24) To answer this question, we have to recall the behaviors asked of the participants in the studies. The participants’ task was to tap their fingers to a metronome in two different conditions. We want to know which nervous system structure would cause the most issues in doing this task if it the structure was damaged. We can go through some general functions of the structures in our answer choices and find the one that would be most important to the synchronized finger tapping. 

  1. Hippocampus. Hippocampus deals with memory, and specifically long-term memory. There is no reliance on memory in the two studies. 
  2. Hypothalamus. Hypothalamus has to do with controlling the autonomic nervous system, which means regulating bodily functions. It controls things like hunger, thirst, sleep, and regulates body temperature and secretion of hormones. This is also not necessary for the two studies.
  3. Cochlea. The cochlea plays a major role in hearing. It functions to transform vibrations of cochlear liquids into a neural signal. This could be a good answer choice because the participant may rely on hearing to synchronize with the experimenter. 
  4. Cerebellum The cerebellum is located underneath the backside of the cerebrum, and governs balance and fine motor movements. Its main function is maintaining coordination throughout the body which is exactly what the participants need in the two studies. This is a more specific and superior to answer choice C. Answer choice D is going to be our best answer. 

25) The sympathetic nervous system controls the body’s automatic response to danger, increasing the heart rate, dilating the blood vessels, slowing digestion, and moving blood flow to the heart, muscles, and brain. It regulates fight-or-flight responses. Essentially, the researcher suggests performing the repetitive task increased sympathetic nervous system activity and influenced the degree of affiliation they felt toward the experimenter. We know this is unlikely based on what we read in the passage. The degree of affiliation was more likely because of synchronizing with the experimenter.

  1. Yes, because studies show that increased sympathetic nervous system arousal can affect how people feel about others. While this observation may or may not be true, the participants engaged in the repetitive activities in both testing conditions. We would theoretically see the same sympathetic activation in both groups. We said in our breakdown of the question, that repetitive task is not what increased sympathetic nervous system activity. This is not a good answer choice.
  2. Yes, because the design used in Study 1 allows researchers to establish the effect of sympathetic nervous system arousal on affiliation ratings. We already established the explanation is not likely to be supported in our breakdown of the question. Reasoning here will be similar to answer choice A. The participants engaged in the repetitive activities in both testing conditions, so we would see the same sympathetic activation in both groups. The repetitive task is not what increased sympathetic nervous system activity. This is not a good answer choice.
  3. No, because performing repetitive behaviors is more likely to increase parasympathetic nervous system activity. While the reasoning may or may not be true, we run into the same problem as answer choices A and B. The participants engaged in the repetitive activities in both testing conditions, so we would see the same parasympathetic activation in both groups. There’s no way we can differentiate like we did in the passage.
  4. No, because both of the conditions involve repetitive behaviors, but participants’ ratings differ under the two conditions. Bingo, this is consistent with our breakdown of the question and the reasoning we used to eliminate the other answer choices. Answer choice D will be our best answer. 

26) We learned from the passage that synchronous actions make participants more prone to comply. We can define the 4 options the test-maker presents us and see which one is consistent with what we read in the passage.

  1. egocentricism. Egocentrism is the inability to consider or understand a perspective other than one’s own. We saw the participants were more likely to comply, but not necessarily understand the perspective of other people.
  2. observational learning. Observational learning or modeling is learning that occurs as a function of seeing, retaining, and, in the case of imitation learning, replicating novel behavior executed by other people. Engaging in synchronous behaviors is not likely to lead to an increase in modeling or observational learning.
  3. obedience. Obedience is form of social influence in which a person yields to explicit instructions or orders from an authority figure. This is similar to compliance which means behavior is influenced by peers. This is going to be the best answer choice here. Engaging in synchronous behaviors is most likely to lead to an increase in social facilitation.
  4. social facilitation. Social facilitation is defined as improvement in individual performance when working with other people rather than alone. This is not what we would see following engaging in synchronous behaviors. Answer choice C is our best answer choice.

Exam 1 P/S Solutions: Questions 27-30

27) To answer this question, we can define residential segregation. Residential segregation is an example of social inequality on the local scale and refers to the separation of demographic groups into different neighborhoods. This can be by race, ethnicity, religion, socioeconomic status, income, and more. It is a product of discrimination that exists in the private real estate market. It has led to less access to resources and opportunities by many marginalized racial groups and low-income individuals. We’re going to define the four options in our answer choices, then we’ll find the type of design that is LEAST appropriate for research on residential segregation.

  1. Ethnographic methods. Ethnographic research studies people or cultures in their own environment, in real social settings. This would actually be a great option for research on residential segregation, but that makes this a bad choice as an answer to this question.  
  2. Experimental methods. In experimental research, variables are manipulated by researchers to compare control and experimental groups or conditions. It would be unethical to have groups of individuals be segregated against for experimental results. This is not an appropriate type of design for research on residential segregation.
  3. Quantitative methods. Quantitative research means collecting and analyzing numerical data. That data can be collected through polls, surveys, and questionnaires which would all be used in researching residential segregation. Answer choice B remains the best option. 
  4. Survey methods. Surveys include collecting information from respondents by asking questions. Reasoning here is going to be similar to answer choice C. Survey information would be helpful for research on residential segregation. We can stick with answer choice B as our correct answer. 

28) Studies will find the mean and median of their quantitative date and it’s quite useful. The mean is also the average of the values you’re dealing with, while median is the middle value. We can use these broad definitions and jump into the 4 statements given as answer choices.

  1. More participants had a BMI over 25 than a BMI under 25. This is factually incorrect because we know the median BMI is 25. That means an equal number of participants have a BMI over 25 and under 25. 
  2. The majority of participants had a BMI between 25 and 30. This is unlikely. We expect the majority of individuals have a BMI over 30 given the median is only 25 but the mean is 30. The only way answer choice B is possible is if a few participants had extremely high BMIs.
  3. Half of the participants had a BMI over 25 and half had a BMI under 25. This is exactly what is meant by a median BMI of 25. This answer choice is consistent with our breakdown.
  4. More outliers among the participants had a BMI under 25 than a BMI over 30. The opposite is likely true because the mean of 30 is quite a bit higher than the median of only 25. If there were outliers, we expect them to have larger BMIs. We can stick with answer choice C as our best answer. 

29) Studying communicable diseases is necessary to learn about the spread of the diseases and to try and combat that same spread between people. To answer this standalone question we can define the four types of analysis listed as answer choices and pick the one that is most likely used to study communicable diseases.

  1. Social support analysis. Social support can be thought of as the soothing impact of friends, family, and acquaintances. It can take many forms, including advice, guidance, encouragement, acceptance, emotional comfort, and tangible assistance. While social support can be beneficial, it’s not likely to be used to study communicable diseases.
  2. Social network analysis. A social network is a social structure between actors, either individuals or organizations. It indicates the ways in which they are connected through various social familiarities, ranging from casual acquaintance to close familial bonds. This is a great way to map relationships between actors and study communicable diseases.
  3. Social stratification analysis. Social stratification is a system of ranking individuals and groups within societies into socioeconomic tiers based on factors like wealth, income, race, education, and power. This has more to do with studying separation between classes, so answer choice B remains superior.
  4. Social reproduction analysis. Social reproduction refers to the emphasis on the structures and activities that transmit social inequality from one generation to the next. This analysis would not be utilized to study communicable diseases. We can stick with answer choice B as our correct answer. 

30) Gentrification occurs when members of the middle and upper classes enter and renovate city areas that have been historically less affluent while the poor urban underclass is forced, by resulting price pressures, to leave those neighborhoods for increasingly decaying portions of the city. Poorer residents are displaced by wealthier newcomers.

  1. Development of affordable housing. This is the opposite of what we’d expect to occur with the process of gentrification. Housing is actually renovated and prices tend to increase, so there is a lack of affordable housing.
  2. Increased neighborhood stratification. Social stratification is a system of ranking individuals and groups within societies into socioeconomic tiers based on factors like wealth, income, race, education, and power. This is exactly what happens with the process of gentrification. We want an effect that is least likely to occur, so answer choice A remains superior.
  3. Displacement of lower-income residents. This is something else we said is likely with the process of gentrification. Poorer residents are displaced by wealthier newcomers. 
  4. Expanded tax base for local government. As wealthier newcomers settle down in the area, average income and the amount of assets that can now be taxed increases. This is also likely to occur with the process of gentrification. We can stick with answer choice A as our best answer choice.

Exam 1 P/S Solutions: Passage 6

31) The test-maker brings up something we can see in Figure 1-adults are more risk averse than adolescents. We’re asked how testing condition affects adults’ scores on the DMQ, but based on something we know from our external knowledge: group polarization. Group polarization refers to the tendency for groups to make decisions that are more extreme than the initial inclination of its members. More extreme does not always mean riskier. A risk averse group will become more risk averse, while a group that typically takes risks will be more willing to take risks.

  1. lower scores on the DMQ in a group than alone. This would be consistent with what we expect from the adults. We already know they are more risk averse than their younger counterparts, so being in a group will only enhance how risk averse they are.
  2. higher scores on the DMQ in a group than alone. This might be true of the adolescents, but the adults are risk averse. Being in a group means that groups will make decisions that are more extreme than the initial inclination of its members. We expect scores to be lower for adults on the DMQ when in a group.
  3. lower scores on the DMQ in small groups but higher scores on the DMQ in large groups. There is no comparison between the size of the group in the question stem, or in the definition of group polarization. We cannot say whether a smaller or larger group will have a larger effect. 
  4. higher scores on the DMQ in small groups but lower scores on the DMQ in large groups. Reasoning here is going to be the same as answer choice C. There is no comparison between the size of the group in the question stem, or in the definition of group polarization. We cannot say whether a smaller or larger group will have a larger effect. We can stick with answer choice A as our best answer. 

32) To answer this question, we can first define behaviorist theory, then we’ll use the information from the passage to tie the definition into our specific topic. 

Which of the following statements is most likely true about the members of a network family?

Behaviorist theory regards an individuals’ actions as ultimately being responses to external stimuli, driven by outcomes. A traditional behaviorist perspective sees outcomes, and not cognition, as the drive behind an individual’s choices and behaviors. Social learning theory believes that personality and behavior are determined by an individual’s cognition about the world around them. In the passage, we saw the results showed a negative correlation between age and scores on the DMQ. We also saw this in Figure 1 and scores from the CG. Let’s see which statement best applies behaviorist theory to explain that relationship. 

  1. have had more experiences in which they have been punished for risky behaviors. This is true. Adults have been alive longer and they’ve had more experiences. Behaviorist theory says that an individual’s choices and behaviors are learned throughout life during interactions with others. An adult that has seen the consequences of risky behaviors is more likely to be risk averse than an adolescent without the same experiences.
  2. are less prone to the effects of peer pressure. Peer pressure is often used to describe instances where an individual feels indirectly pressured into changing their behavior to match that of the members of their peer group. This is out of scope in this situation because we’re focused on the behaviorist theory.
  3. have a more mature prefrontal cortex that inhibits impulsive behavior. This is the biological perspective. This perspective focuses on the role of genetics and the brain in shaping personality.
  4. have a more developed superego that inhibits the impulses of the id. This is the psychoanalytic approach of personality which argues that human behavior is the result of the interactions among three component parts of the mind: the id, ego, and superego. We can stick with answer choice A as the best answer to this question.

33) This question is focused on the DMQ measure in the passage. The results showed a negative correlation between age and scores on the DMQ, but we want to know how we can support the validity of these results. The DMQ is a decision-making questionnaire, so there is self-reporting taking place. There is the possibility of the results not being valid. For example, the participants may show social-desirability bias and report answers that they think are the most socially acceptable. This is why we should check with the scores from the CG and compare. The CG measure is a behavioral measure, unlike the DMQ measure.

  1. A positive correlation between the DMQ scores and the CG scores This is not only something we see from the information in the passage, but it’s also a way to support the validity of the DMQ measure. By comparing DMQ and CG scores, the researchers have results from self-reporting, but also a behavioral measure. This supports the validity of the DMQ measure.
  2. A negative correlation between the DMQ scores and the CG scores. This would have the opposite effect of what we’re looking for. For both the DMQ and the CG, higher scores indicated riskier behavior. If we have a negative correlation between the two scores, that weakens the validity of the DMQ measure. 
  3. A positive correlation between participants’ scores on two different administrations of the DMQ. This is something that is done to enhance the reliability of scores, but this does not make the scores more valid. Best way to support the validity in this case is to compare with the behavioral measure (CG) score.
  4. A negative correlation between participants’ scores on two different administrations of the DMQ. This would tell the researchers their measured scores are unreliable, but would not support the validity of the DMQ measure. We can stick with answer choice A as the best, most relevant answer choice.

34) Like so many of the behavioral questions you’ll see this relies on knowing your vocabulary and your content. Erikson’s’s Stages of Psychosocial Development are based on (and expand upon) Freud’s’s psychosexual theory. Erikson proposed that we are motivated by the need to achieve competence in certain areas of our lives. According to psychosocial theory, we experience eight stages of development over our lifespan, from infancy through late adulthood. 

Generativity vs Stagnation is relevant to adults. When people reach their 40s, they enter the time known as middle adulthood, which extends to the mid-60s. The social task of middle adulthood is generativity vs stagnation. Generativity involves finding your life’s work and contributing to the development of others through activities such as volunteering, mentoring, and raising children. The only option of the three that would be dealing with generativity vs stagnation is option III. That means we can stick with answer choice B as our best, correct answer.

Exam 1 P/S Solutions: Passage 7

35) While this is a passage-related question, we can treat it like a standalone question that is testing our knowledge of assimilation. We can define assimilation here, then we’ll jump into the four statements that make up our answer choices. 

After moving from one culture to another, it takes time to learn a new set of cultural norms and acceptable behaviors. The process to acclimate to a new culture is called assimilation. Assimilation can occur in a variety of ways, including language acquisition and learning about the social roles and rules of the newly adopted culture. As we go through out answer choices, keep in mind our correct answer does NOT identify an aspect of assimilation.

  1. Assimilation is the influence that cultural changes have on an individual’s health. Assimilation does not have to do with influence or changes to an individual’s health. Based on what we read in the passage, it is possible assimilation will eventually have an influence on health, but that is not describe assimilation itself.
  2. Assimilation is the process of cultural adaptation that results from geographic mobility. This is consistent with assimilation. We talked about this in our breakdown as individuals will take time to adapt as they move; geographic mobility can often mean changes in cultural normal and acceptable behaviors.
  3. Assimilation occurs when individuals adopt the cultural norms of a dominant culture. This is also consistent with our breakdown of assimilation. I mentioned it takes time to learn a new set of cultural norms and acceptable behaviors. The process to acclimate is assimilation.
  4. Assimilation occurs when individuals relinquish the cultural norms of their childhood. This is also a true statement about assimilation. Individuals will adopt new cultural norms and behaviors that are different than those of their childhood. We can stick with answer choice A as the only option that does NOT identify an aspect of the concept of assimilation. 

36) This is another vocabulary-heavy question. We’re given four concepts as answer choices which means we’ll define each one in just a second. We want a concept that’s consistent with the benefits of bilingualism being related to the idea that structure of language affects the perceptions of its speakers. 

  1. Weber’s Law. Weber’s law states that the difference threshold is proportional to the magnitude of the stimulus. This relates the difference in threshold with the initial stimulus intensity, which is out of scope for this question.
  2. The nativist hypothesis. Nativist hypothesis states that children have the innate biological ability to learn a language. This is also out of scope in this situation. 
  3. Schacter–Singer theory. According to the Schacter–Singer theory, emotion results from the interaction between two factors: physiological arousal and cognition. The theory claims that physiological arousal is cognitively interpreted within the context of each situation, which ultimately produces the emotional experience. This is out of scope.
  4. Linguistic relativity. The theory of linguistic relativity states that the structure of a language influences the way its speakers conceptualize the world. This is exactly what we’re looking for in our question stem; this question came down to knowing vocabulary terms. Language influences a speaker’s perception of the world. Answer choice D is our correct answer.

37) In the passage we learned that researchers designed a study to investigate the relationship between language and self-rated health. The researchers’ statistical models considered multiple variables including socioeconomic status (SES), assimilation, family and social support, stress and discrimination, and healthcare access and health behaviors. Researchers found associations between bilingualism and good health that are partially explained by SES and family and social support. If Mexican immigrants did not have the same SES or family/social support as the other Latino immigrant groups, we might not see the same, good health. 

  1. Cuban and Puerto Rican immigrants have higher average SES than Mexican immigrants. This is a strong possibility based on what we read in the passage. Researchers found associations between bilingualism and good health that are partially explained by SES and family and social support. If Mexican immigrants did not have the same, high SES as Cuban and Puerto Rican immigrants, they would not see the same good health status.
  2. Mexican immigrants have more extensive social support networks than Cuban immigrants. This is the opposite of what we’d expect. We learned in the passage that there are associations between bilingualism and good health that are partially explained by family and social support. We expect the Cuban immigrants that reported superior physical and mental health had stronger social support.
  3. Mexican immigrants report greater levels of assimilation than Puerto Rican immigrants. The passage did mention in the first paragraph that assimilation is associated with worse health. However, later in the passage the author mentions, “the researchers did not find evidence that assimilation, stress and discrimination, or healthcare access and health behaviors affected the language–health relationship.” Answer choice A remains superior.
  4. Cuban and Puerto Rican immigrants report higher stress levels than Mexican immigrants. This is going to tie into our explanation for answer choice C. The author says, “the researchers did not find evidence that assimilation, stress and discrimination, or healthcare access and health behaviors affected the language–health relationship.” Furthermore, stress is normally considered a negative in relation to health. We can stick with answer choice A as our best answer. 

38) This is a passage-related question that we can almost treat like a standalone question. We’re going to define social support then find the one answer choice that is not a health benefit of social support. Social support can be thought of as the soothing impact of friends, family, and acquaintances. It can take many forms, including advice, guidance, encouragement, acceptance, emotional comfort, and tangible assistance.

  1. friends and family members that help patients adhere to medical treatment. This is a great start. Friends and family members helping patients adhere to medical treatment are helping the health of the patient. This is considered social support, and there is a clear health benefit. We’re looking for an answer that is not a health benefit.
  2. friends and family members that help patients to reduce harmful behaviors. This is similar to answer choice A. Reducing harmful behaviors is beneficial to one’s health. Family and friends that do that are part of social support. This is also not a good answer to this specific question. 
  3. social relationships that help individuals to cope with stressful events. This is consistent with social support; coping with stressful events is a good thing for an individual’s health. This is a health benefit of social support, so not a good answer to this specific question.
  4. social activities that help individuals to avoid loneliness and boredom. Social activities do not necessarily fall under our definition of social support. We’re looking for the soothing impact of friends, family, and acquaintances. While this can be beneficial to an individual, this does not explicitly show the health benefits of social support specifically. We can stick with answer choice D as our best answer.

Exam 1 P/S Solutions: Passage 8

39) To answer this question, we can define working memory then decide the length of sequence the adults would be able to replicate. Working memory is the ability to hold information for a short duration of time (on the order of seconds). In the process of encoding, information enters the brain and can be quickly forgotten if it is not stored further in short-term memory. When several elements (such as digits, words, or pictures) are held in short-term memory simultaneously, their representations compete with each other for recall or degrade each other. When we’re trying to quantify the number of taps of the participants were able to replicate, we use George Miller’s Magic Number of 7±2. This number represents the number of objects a human can hold in short-term memory and is going to be a good estimate for the number of taps the participants can replicate. This is a quantitative value where we did no approximating or rounding. We can pick the only answer that matches our predicted value: Answer choice B.

40) To answer this question, it is important to distinguish between subjectivity and objectivity. Subjectivity is based on feelings, emotions, and opinions. There is no unarguable fact supporting subjectivity. Objectivity is truth that is independent of subjectivity-meaning no bias, prejudice, or partiality. We want to find a subjective measure of motivation in our four answer choices.

  1. The number of grid sequences participants attempted to solve on the computer task. This is an objective measurement. The number of grid sequences is not affected by feelings, emotions, or opinions. 
  2. The participants’ decision to continue playing the game or stop to read a magazine. This is once again an objective answer choice. The decision is either play the game or read a magazine. There’s no emotion or opinion involved in that final decision.  
  3. Participants’ answers on the exit questionnaire. We recall from the passage that participants answered an exit questionnaire asking how much they liked the task and whether they would like to have it at home. The answers to this question involve feelings, emotions, and opinions. In other words, these answers will be subjective. This is going to be our best answer choice so far. 
  4. Participants’ time spent not playing the game using the computer mouse. This measurement can be done with a timer without any feelings, emotions, or opinions. The time spent not playing the game using the computer mouse is an objective measurement. We can eliminate this answer choice as well. We’re left with answer choice C as our best answer.

41) Many behavioral questions revolve around knowing the definition of vocabulary words and phrases. To answer this question, we can start by going over extrinsic factors. Motivations can be either intrinsic (arising from internal factors) or extrinsic (arising from external factors). Incentive theory argues that people are primarily extrinsically motivated—meaning that most motivations stem from extrinsic sources. Extrinsically motivated behaviors are performed in order to receive something from others, or avoid certain negative outcomes. Tying this back to our passage, the question asks us to consider the study in the passage and pick which extrinsic factor more likely accounts for the motivational differences between the groups. The final two paragraphs of the passage go over how motivation was assessed, and how participants fared in the different situations. We can focus on these paragraphs and decide the extrinsic factor that most likely accounts for motivational differences. 

  1. Rewards and losses tied to memory performance for the gaming group. This checks all of the boxes we need for an extrinsic factor that accounts for the motivational difference. A rewards and losses are external factors and they function to motivate the participants as they go through the gaming task. We were told Points were earned by responding correctly, which strengthened participants’ efforts and weakened animated opponents. Points were lost by responding incorrectly, strengthening the opponents’ power. This is consistent with what we’re looking for in a correct answer.
  2. The presence of animated characters in the visual display for the gaming group. This is not something that is being used to motivate participants. If the animated characters only showed up after a correct answer, this might be a viable answer. However, the presence alone is not enough to be a good answer here. Answer choice A remains superior.
  3. The opportunity to stop playing the game for the gaming and non-gaming groups. Reasoning here is going to be similar to answer choice B. The opportunity to stop playing the game is not something that is being used to motivate participants. We’re looking for an extrinsic factor that would account for the motivational differences between groups, so answer choice A remains superior.
  4. The opportunity to state they disliked the game for the gaming and non-gaming groups. This is similar to answer choice C in that the opportunity to state they dislike the game is not something used to motivate participants. Once again, we’re looking for an extrinsic factor that would account for the motivational differences between groups. The only answer choice that does this is answer choice A.

42) We can recall the experimental setup from the passage. We were told in the non-gaming condition, squares on a four-by-four grid lit up in random order on a screen. This question stem wants us to visualize the squares that are lit up and explain the process of organizing the top row as a single entity. We also had to use external knowledge earlier in this question set to define working memory. Working memory is the ability to hold information for a short duration of time.

  1. Divided attention. Divided attention is the ability of the brain to attend to two different stimuli at the same time. The brain responds to multiple demands of the environment at the same time by using simultaneous attention, allowing a subject to process different information sources and carry out multiple tasks at one time. This is not in play in the scenario presented in the question stem. 
  2. Priming. Priming involves the exposure to a stimulus that influences a response (that is not conscious) to a subsequent stimulus. This is not what is happening in the question stem. The participants are organizing the sequence into a whole. 
  3. Automatic processing. Automatic processing involves processes that are quick and initiated involuntarily and automatically without active control. This is not what we’re looking for in this situation based on what we read in the passage. We can keep comparing. 
  4. Chunking. Chunking is the process of organizing parts of objects into meaningful wholes. The whole is then remembered as a unit instead of individual parts. Examples of chunking include remembering phone numbers or words, but also what we saw in the question stem with the four squares lighting up in a row. This is going to be our best answer. 

43) Despite being part of a question set related to a specific passage, this is predominantly going to rely on our external knowledge. In the first computer task, squares on a four-by-four grid lit up in random order on a screen. Prior to working memory being activated, we use sensory memory. 

Sensory memory allows individuals to retain impressions of sensory information for a brief time after the original stimulus has ceased. It allows individuals to remember great sensory detail about a complex stimulus immediately following its presentation. Sensory memory is an automatic response considered to be outside of cognitive control. Information from sensory memory has the shortest retention time, ranging from mere milliseconds to five seconds, and it is retained just long enough for it to be transferred to working memory.

  1. Implicit memory. Implicit memories are long-term memories that are not consciously remembered, including procedural memories and emotional conditioning. This is not what we’re looking for in this situation.
  2. Sensory memory. This answer choice matches our breakdown. Sensory memory is retained just long enough for it to be transferred to working memory. This is going to be our best option.
  3. Short-term memory. Short term memory and working memory are used interchangeably. We know from our external knowledge and the question stem this will activate after sensory memory. We can eliminate this answer choice.
  4. Episodic memory. Episodic memory is a kind of explicit memory, that includes specific events/episodes. This is not consistent with the answer we’re looking for (sensory memory) or the timeframe presented in the question stem. We can stick with answer choice B as our best option.

44) The passage tells us the children assigned to the tasks were between 7 and 12 years of age with ADHD. This question is basically asking us what conclusions we can make after reading about the studies in the passage. To answer this question, we can go through each of the four conclusions listed as answer choices and find the one that is most likely.

  1. Immediate performance feedback reduces impulsive responding in children with ADHD. The author tells us: in the gaming condition, animated figures were added. Points were earned by responding correctly, which strengthened participants’ efforts and weakened animated opponents. Children were given immediate feedback. The author also tells us the results indicated that participants in the gaming condition spent more time playing the game, attempted to replicate more sequences, and made fewer errors.
  2. Exposure to a gaming element increases impulsive responding in children with ADHD.  We know from the passage that participants in the gaming condition spent more time playing the game, attempted to replicate more sequences, and made fewer errors. This is the opposite of what answer choice B suggests.
  3. Gaming elements on working memory tasks place high demands on divided attention and inhibit focus. Divided attention is the ability of the brain to attend to two different stimuli at the same time. This is out of scope as the author does not give us details about divided attention in the passage. Gaming elements also increased focus, not inhibit focus. 
  4. Low levels of intrinsic motivation in children with ADHD are minimally enhanced with external reinforcers. This is similar to answer choice C because we’re looking for an answer that is correct based on the findings in passage. Low levels of intrinsic motivation are not minimally enhanced. The children actually improved, and the author says the same participants were also more likely to report liking the game and wanting a version to play at home. External reinforcers enhanced motivation quite a bit. We can stick with answer choice A as our best answer. 

Exam 1 P/S Solutions: Questions 45-48

45) This is a standalone question that relies on knowing different study methods. Instead of knowing a single study method, we’re asked to go through four answer choices to find the methodological approach that is most appropriate to gain insight into how a social movement organization presents its beliefs to the general public in an effort to attract new members. This is a broad, open-ended question so we’ll jump right into our answer choices.

  1. Observing meetings about the organization’s marketing budget. While meetings about the organization’s marketing budget might be important to the running of the organization itself, this is not how the organization presents its beliefs to the general public. This meeting is most likely to provide internal information, and not meant to attract new members from the public. 
  2. Conducting a content analysis of the organization’s website. The content on an organization’s website is crucial to attracting new members, especially in this day and age. A website is often the first point of contact between an organization and prospective members, so it is important to know how the organization is presenting its beliefs to the public. This is going to be an appropriate approach. 
  3. Surveying organization members about their beliefs. This would only tell us about the beliefs of individual organization members and not about how the organization presents beliefs to the general public. This would not be a way to attract new members. Answer choice A remains superior.
  4. Randomly sampling the general population about social movement organizations. This answer choice does not mean give insight into how a specific organization presents its beliefs to the general public. The general population is not going to be responsible for the public presentation of beliefs by an organization. Only answer choice B lists a way to gain insight into how the organization presents its beliefs in an effort to attract new members. Answer choice B is our correct answer.

46) The author gives us some background about a specific immigrant teen. We’re asked to categorize the teens of the new country’s dominant culture for the immigrant teen. To answer this question, we’ll define the four vocabulary terms given as possible answers and find the one that best defines the teens. 

  1. Assimilated group. This is likely just added as a distractor and not something we see on AAMC’s content outline. Assimilation is the process by which an individual or group becomes part of a new culture. The immigrant team is working to assimilate, but assimilated group is not a term we see in sociology. 
  2. Reference group. A reference group refers to a group to which an individual or another group is compared. The immigrant teen sees the behaviors of the teens in the new country and wants to modify his own behaviors to match this dominant culture. The teens of the new country’s dominant culture are a reference group for the immigrant teen.
  3. Majority group. A majority group can be a generic term for the dominant ethnicity in a population or, in this case, country. We’re not told about the specific ethnicity of the teens in the new country, but we do know these teens had an influence on the immigrant teen. These teens served as a reference group for the teen, not just a majority group. Answer choice B is a superior answer for the specific question being asked.
  4. Peer group. A peer group is a social group whose members have interests, social positions, and age in common. The influence of the peer group typically peaks during adolescence. However, peer groups generally only affect short term interests unlike the family, which has long term influence.

47) The researchers here are conducting an experiment to note the effects of scent on viewers’ ratings of attractiveness of paintings. In experimental research, variables are manipulated by researchers to compare control and experimental groups or conditions. 

There are typically independent and dependent variables in every experiment we come across. We typically need to identify both, and more importantly, we have to understand their relationship. An independent variable can be controlled or changed during a study-this variable(s) is independent of the other factors of the experiment. In this case, that is the scent being added to the rooms or not. As the independent variable is changed, we’re tracking the dependent variable to see the effect of the change in the independent variable. In this case, we’re seeing the change in viewers’ ratings of attractiveness of the paintings. So, the independent variable varies, and we track the change on the dependent variable. In the case of this experiment, the independent variable is the exposure to stress. We want to go through our four answer choices and find the one that is accurate about the study.

  1. The study’s design is an observational study since viewers were observing paintings and noting their ratings. The study is not an observational study. In an observational study, the independent variable is not changed by the researchers (as is the case in this situation). This particular study is actually controlling the scents being added to one of the rooms with paintings to see the influence on viewers’ ratings.
  2. The independent variable is the presence or absence of scent in the rooms. This is consistent with our breakdown. An independent variable can be controlled or changed during a study. This variable is independent of the other factors of the experiment. In this case, that is the scent being added to the rooms or not being added to the rooms.
  3. The dependent variable is the amount of time viewers spent observing paintings. The dependent variable is tracked to see the effect of the change in the independent variable. In other words, if scent is added to one of the rooms, we’re seeing the change in viewers’ ratings of attractiveness of the paintings. The dependent variable is the viewers’ rating.
  4. Informing viewers ahead of time whether a scent was absent or present in the room is important for accurate ratings. Telling the viewers that they are being studied and tested can actually influence the results. This is not necessary for accurate ratings. We can also eliminate this answer choice. Only answer choice B is accurate regarding the study.

48) People often say that riding a bike is something you never forget how to do. Of course, to get to that point, you have to learn how to ride the bike initially. That initial process can be tough (and painful!) but once you learn, you’re able to rely on your memory to ride the bike easily. To answer this question, we’ll define the four types of memory listed and find the one that is used when riding a bike.

  1. Semantic. Explicit memories include general knowledge about the world. We can consciously remember or recall these memories. Riding a bike does not have to do with knowledge or facts. 
  2. Episodic. Episodic memory is a kind of explicit memory that includes specific events/episodes. These memories can be consciously remembered or recalled as well. This is also not what we use when riding a bike. 
  3. Explicit. The two types of memories we’ve talked about can be called explicit memories. Explicit memories are long-term memories that are consciously remembered. Riding a bike does not involve consciously remembering.
  4. Procedural. Unlike the other three options, implicit memories are less conscious, and include procedural memories like how to ride a bike. We can perform certain functions without conscious awareness. Riding a bike requires procedural memory; answer choice D is our best answer.

Exam 1 P/S Solutions: Passage 9

49) This is a big picture question that is going to rely on us recalling the role of cultural liaisons from the passage, but also using external knowledge to define the four phenomenon listed as our answer choices. The author details the differences in health outcomes at the macro- and micro- levels. We’re told that, at the micro-level, the quality of the physician–patient interaction is one of the strongest predictors of healthcare quality and treatment outcomes. We’re told race concordant physician–patient relationships result in higher levels of patient trust and physician attention. In order to mediate this dynamic, specially trained cultural liaisons who work with both patients and physicians have been introduced into many clinics. We can use this information and go through our answer choices.

  1. Network redundancy because of the amount of people with whom they come into contact. Network redundancy refers to interacting with individuals of the same/similar network. For example, if you meet a set of best friends and speak to them individually, a lot of the social information you get from the friends will be redundant. They are interacting with one another and offer similar social information. Physicians and patients don’t always have the same network, so this is not likely to be an issue for the cultural liaisons.
  2. Homogeneity because they will see the same patients over and over. Homogeneity involves doing multiple roles in a community. Seeing the same patients repeatedly does not cause the cultural liaisons to experiences homogeneity. They are seeing these patients in the same role every time.
  3. Role strain because of their collegiality with physicians and their responsibility to patients. Role strain is the stress or strain experienced by an individual when incompatible behavior, expectations, or obligations are associated with a single social role. This is consistent with what we read in the passage. The cultural liaison has to exhibit a certain behavior around the physicians, but also advocate for patients. This is going to be our best answer. 
  4. Role conflict because of their status as a non-medical professional in a clinic setting. Role conflict describes the conflict between or among the roles corresponding to two or more statuses held by one individual. The cultural liaison only has one role in this case, but has to work with both physicians and patients. Answer choice C remains the best answer choice.

50) This is going to come straight from our external knowledge. The author brought up in the passage: “Race concordant physician–patient relationships result in higher levels of patient trust and physician attention.” We’re going to use our external knowledge to provide a reason.

  1. both the physician and patient belong to the same in-group. In-groups are social groups to which an individual feels he or she belongs. In-group bias is a phenomenon in which people tend to act more favorably towards people who they perceive to be part of their in-group. The racial concordance (shared identity/background) is consistent with the physician and patient belonging to the same in-group. 
  2. the physician and patient share similar levels of status. While this might be the case when physicians and patients have the same social status and class, there is no guarantee people of the same race will share similar levels of status. Answer choice A remains superior.
  3. of racial hostility present in race-discordant relationships. This is out of scope. There is no level of racial hostility brought up in the passage. The author gave the example that black patients report having less input in health decisions and lower levels of physician trust if their race differs from that of their physician. That does not mean hostility. We can stick with answer choice A as our best answer.
  4. of the lack of role strain experienced by the physician. Role strain is the stress or strain experienced by an individual when incompatible behavior, expectations, or obligations are associated with a single social role. A physician is not necessarily going to change their behavior or obligations from patient to patient. There is no evidence of role strain because of patients from different races. We can stick with answer choice A as our correct answer. 

51) This is going to be another passage-related question that comes down to using our external knowledge. We’re told in the passage that, at the micro-level, the quality of the physician–patient interaction is one of the strongest predictors of healthcare quality and treatment outcomes. Race concordant physician–patient relationships result in higher levels of patient trust and physician attention. In order to mediate this dynamic, specially trained cultural liaisons who work with both patients and physicians have been introduced into many clinics. We want to know the benefit of this liaison as opposed to training physicians in cultural competency.

  1. Communicating across racial and cultural lines is difficult and requires a trained specialist to do it effectively. This may or may not be true, but we cannot say this is an explicit advantage of having a specially trained cultural liaison because the author does not talk about this in the passage. This answer choice is out of scope.  
  2. Physicians are inherently resistant to cultural training because of their medical biases. There is no evidence in the passage that talks about physicians being inherently resistant to cultural training. This would be a much deeper issue if this were true.
  3. Adding a cultural liaison creates a triadic group, a more stable arrangement than a dyadic group. Dyads and triads are the smallest social groups. Social interaction in a dyad is typically more intense than in larger groups because neither member shares the other’s attention with anyone else. A triad is more stable than a dyad because one member can act as a mediator should the relationship between the other two become strained. In this case, the triad being more stable makes this a great answer choice.
  4. Larger groups facilitate the transfer of information more effectively than smaller groups. This is factually incorrect. Smaller groups actually facilitate the transfer of information more effectively than larger groups. We can stick with answer choice C as our best answer.

52) First thing we can do as we approach this question is reference where the author talks about neighborhood-level socioeconomic inequalities. We’re told racial and ethnic minorities are more likely to live in environments characterized by neighborhood disorder. The author also says the negative effects of segregated neighborhoods contribute to the growing social distance between residents of lower-socioeconomic neighborhoods and those living in higher-socioeconomic environments. That means these patients will have less in common with their physicians in terms of class, cultural background, and social status.

  1. Status hierarchies rely on socioeconomic differences above other master status characteristics. A master status is a status that dominates over the other statuses you might own. In other words, it’s the defining characteristic of an individual. We want to reference the passage for this answer choice. The author says, “The social distance hypothesis suggests that patients who have more in common with their physicians in terms of class, cultural background, and social status receive more attention from physicians. Primarily, this has been attributed to differences in communication styles and cultural competencies.” We know socioeconomic differences can be a big influence on physician-patient interactions, but the social distance between patients and physicians is going to be the biggest potential issue.
  2. Physical boundaries create social boundaries and closed networks which develop their own cultures. This is something we went over in our breakdown. We’re told the negative effects of segregated neighborhoods contribute to the growing social distance between residents of lower-socioeconomic neighborhoods and those living in higher-socioeconomic environments. These boundaries lead to closed networks which develop their own cultures, so the physician and patient cannot fully relate to one another. This is a great answer choice.
  3. Socioeconomic inequalities create role conflict in physicians who are trained to treat all patients equally. Role conflict describes the conflict between or among the roles corresponding to two or more statuses held by one individual. The physicians in this case only have one role (physician), despite having patients of different backgrounds. Answer choice B remains superior.
  4. The physician–patient relationship is not subject to the same external forces that govern other relationships. We know this is not the case. For example, black patients report having less input in health decisions and lower levels of physician trust if their race differs from that of their physician. Additionally, race concordant physician–patient relationships result in higher levels of patient trust and physician attention. These relationships are governed by the same external forces, and are just as important. Answer choice B is going to be our best answer.

Exam 1 P/S Solutions: Passage 10

53) This was a relatively short passage that revolved entirely around inequality and discrimination in the workplace. There were some key differences between the first part of the passage and the last part of the passage. First part of the passage talks about discrimination and inequality and gets into specific quantitative data. The second part of the passage jumps into some deeper analysis to examine how inequality in the workplace is created and maintained. We can define the four descriptions listed as possible answer choices and see which one best applies to the research in the passage.

  1. qualitative study. Qualitative studies focus on non-numerical data to understand something like we see in the second paragraph of the passage. The researchers used quantitative research in paragraph 1, but paragraph 2 focused on an in-depth analysis of claims from 740 cases. This is a decent answer because the second part of the passage described a qualitative study.
  2. quantitative study. Quantitative research means collecting and analyzing numerical data. That data can be collected through polls, surveys, and questionnaires. The first paragraph of the passage used quantitative research, which means this answer is also a decent one. Both answer choice A and B mention a type of research that was used in the passage. 
  3. mixed-methods study. This is going to be our best answer for the time being. We already mentioned that Both answer choices A and B mention a type of research that was used in the passage. A mixed-methods study is exactly what its name suggests: there’s a mix of two methods-quantitative and qualitative. This is going to be our best answer.
  4. case study. Case studies are much more focused studies that focus on single individuals or groups, not larger samples like we see in the passage. We can eliminate this answer choice as well. Answer choice C is going to be the correct answer.

54) First thing we’re going to do to answer this question is actually define the conflict theory perspective. From there, we can relate to status hierarchies. Conflict theory is a way of studying society by focusing on inequality between different groups and sees social life as a competition and focuses on the distribution of resources and power between these groups. Conflict theory views change as abrupt and revolutionary; that change and revolution is the result of inequality. In the conflict perspective, change comes about through conflict between competing interests, not consensus or adaptation. Let’s look through the four possible answer choices and find the one that is consistent with the conflict theory perspective. A key thing we want to remember is the last finding in the paragraph.

  1. efficiency in the organization, whereas discrimination is a byproduct of these structures. There is nothing in the passage or our external knowledge that suggests that status hierarchies facilitate efficiency in an organization. Discrimination is a byproduct of preset cultural views and procedures/rules that allow supervisors to act on stereotypes. 
  2. the preservation of structural power while being maintained by practices of discrimination. Typically, when we see conflict theory, we want an answer choice that talks about inequality and the distribution of resources and power. Groups will maintain their power as long as possible, often through discrimination.
  3. effective social interactions, whereas discrimination is a subjective quality of those interactions. This answer choice sounds like interactionism. Interactionism focuses on interactions between individuals and how they affect social processes. We’re looking for an interpretation that is instead consistent with the conflict theory perspective. Answer choice B remains superior. 
  4. the necessary operation of the workplace while removing ineffective members by practices of discrimination. This has more to do with functionalism. Functionalism is a framework for building theory that sees society as a complex system whose parts work together to promote solidarity and stability. Answer choice B is going to remain the best answer choice.

55) This is a very broad and open-ended question. In order to answer this question, we’ll consider the findings from the passage and find the hypothesis that is most likely to be supported. For questions like these, it is often beneficial to think about the key findings and any results given to us in the passage, which are likely going to come from the final paragraph in the passage.

  1. Regular employees with stereotypical views would exclude minority group members, regardless of organizational culture. The author tells us gatekeeping actors operate within an organizational structure where procedures and rules allow them the flexibility to act on stereotypes. Without that necessary organizational culture in place or preset cultural views, we can’t say minority group members would be excluded. We also cannot say regular employees would exclude these group members or not based on what we read in the passage.
  2. Workplace supervisors with stereotypical views would exclude minority group members, regardless of organizational culture. This ties into answer choice A and our reasoning there. We’re told gatekeeping actors operate within an organizational structure where procedures and rules allow them the flexibility to act on stereotypes. Without that necessary organizational culture in place or preset cultural views, we can’t say minority group members would be excluded. 
  3. Organizational culture is the primary determinant of whether minority populations are excluded from a workplace. Organizational culture can be a determinant of whether minority populations are included, but you need workers with stereotypical views in order for minority populations to be excluded from a workplace. If supervisors and employees do not have stereotypical views, culture alone likely won’t exclude minority populations from a workplace.
  4. The interplay between a supervisor’s stereotypical views and an organization that fosters those stereotypes would be more detrimental to minority group members. This is going to combine the two necessary ingredients to make working in an organization most detrimental to minority group members. It’s possible to have a supervisor with stereotypical views or an organization that fosters stereotypes. However, the combination of the two is more detrimental to minority group members. Answer choice D is the most likely to be supported.

56) Something I want you to note is, this could be a question in a huge portion of the passages you’ll see on the exam. It’s imperative you understand the studies you come across because the test-maker can give you questions that show you understand the purpose, design, variables, results, and conclusions of any study or experiment. For this particular question, we’re going to go through the four conclusions listed and find the one that is best supported based on the passage. That means external knowledge may play a role, but what we read in the passage is going to have to support the conclusion.

  1. Gender discrimination is more likely to occur than racial discrimination. The key to this answer choice is in Table 1. We have a large sample size of both race and gender claims. There are more race claims in Sample 1. I want to note that the percentages we see in each column are very similar as well. We cannot conclude that gender discrimination is more likely to occur than racial discrimination based on this passage.
  2. Race and gender discrimination manifest in similar ways. This is going to rely on Table 1 and the data the researchers got from looking at Sample 1. The percentages we see in each column are very similar, meaning discrimination is manifesting in the same relative percentages across the five categories. This is going to be superior to answer choice A.
  3. The incidence of race and gender discrimination is equivalent. We cannot say this statement is true based on what we see in the passage. We have a large sample size of both race and gender claims, and there are more race claims in Sample 1. We see the percentages of each category are similar, but they are not exactly equivalent. The verbiage here is going to make it so answer choice B remains superior.
  4. Minority women are especially subject to discrimination. This may or may not be true, but it cannot be supported by what we read in the passage. We know minorities and women are both subject to discriminatory practices. However, we cannot say with any certainty that minority women are especially subject to discrimination. We can stick with answer choice B as our best answer choice. 

Exam 1 P/S Solutions: Questions 57-59

57) This is a standalone question and for the behavioral section of the exam, that usually means we’ll rely on vocabulary terms. This question should be no different. We can define the bystander effect, then we can go over dependent variables and find our best option. The bystander effect refers to cases where individuals do not offer any means of help to a victim in an emergency when they believe that others are present. The probability of helping victims is inversely related to the number of bystanders; the greater the number of bystanders, the less likely it is that anyone of them will help. Therefore, a single student would be most likely to help, while many students would mean it’s less likely anyone helps. 

An independent variable, just like its name suggests, can be controlled or changed during a study-this variable(s) is independent of the other factors of the experiment. As the independent variable is changed, we’re tracking the dependent variable to see the effect of the change in the independent variable. So, the independent variable varies, and we track the change on the dependent variable. We want to identify a possible dependent variable. 

  1. The time of day that the experiment is carried out. The time of day the experiment is carried out can be controlled by the student investigating the bystander effect. This can be classified as an independent variable.
  2. The number of people present in the cafeteria. The number of people present in the cafeteria is an independent variable. As the number of people varies, we should see a difference in the likelihood someone helps a student in need. 
  3. The amount of time a student takes to assist another student in distress. This answer choice lists a dependent variable. We mentioned the probability of helping victims is inversely related to the number of bystanders. As the number of bystanders varies, we want to measure the amount of time a student takes to assist another student in distress. This is going to be the best answer so far. 
  4. The temperature in the cafeteria. This is also an independent variable that is not typically associated with the bystander effect. We’re looking for a dependent variable here, so we can stick with answer choice C as our best answer.

58) This is a standalone question that relies on us knowing our vocabulary terms. In this particular question, we’re given a description and we’re matching the research methodology from our four answer choices. To answer this question, we can define each of the four methods listed and find the one that best matches the description in the question stem.

  1. Comparative methods. Comparative research, like the name suggests, functions to compare two or more subjects. This allows researchers to note similarities and differences, and also make discoveries about the subjects being compared. In our question stem, we’re told we want systematic observation of a complete social environment, not necessarily a comparison between subjects.
  2. Ethnographic methods. Ethnographic research is research that studies people or cultures in their own environment. This is exactly what we’re looking for in the question stem. We can stick with answer choice B as our correct response for the time being.
  3. Experimental methods. Experimental research is research in which scientists manipulate variables to test the difference between an experimental and control group. The method described in the passage is an observational, not experimental method. Answer choice B remains superior in this situation.
  4. Survey methods. Surveys include collecting information from respondents by asking questions. There are no surveys being conducted here according to our question stem. There is only systematic observation of a social environment. We can eliminate this answer choice and stick with answer choice B as our best answer.

59) Similar to the other standalone questions in this question set, this question relies on knowing our vocabulary. We’re provided a definition here, and we’re going to match it to one of the four terms given to us as potential answer choices. We’re going to use the same method we always do for these questions where we define the four answer choices and find the definition that is most consistent with the question stem. 

  1. Class systems. A class is usually synonymous with socioeconomic status, which is one’s social position as determined by income, wealth, occupational prestige, and educational attainment. These can be changed to an extent throughout one’s life, and allow for social mobility. 
  2. Caste systems. The existence of caste systems is a significant factor that prevents social mobility. The caste system in ancient India was used to establish separate classes of inhabitants based upon their social positions and employment functions in the community. This position is inherited and there is no room for social mobility.
  3. Economic systems. An economic system is related to the production, resource allocation, and exchange and distributions of goods and services in a society. This is not going to directly affect social mobility. 
  4. Financial systems. This answer is similar to answer choice C. A financial system permits the exchange of funds between market participants. This is not related to closed status positions that hinder social mobility. Only answer choice B matches the definition in our question stem.

Which of the following best describes the term family?

The current-day definition of family refers to two or more individuals who depend on one another for emotional, physical, and/or financial support.

Is a broad concept that refers to marriage between two people who are similar to one or more characteristics?

Homogamy is the marriage or other relationship between people with similar characteristics, socioeconomic status, and ethnicity. Economists call assortative mating what sociologists call homogamy.

When a person has fewer intimate relationships than he or she desires it is referred to as?

Terms in this set (79) When a person has fewer intimate relationships then she desires, this is referred to as: social loneliness.

Which of the following best describes the term cohabitation?

Which of the following best describes the term "cohabitation"? Living with someone in an intimate, sexual relationship without being legally married.